What is the solution to X+9 = 24?
A. x = 33
B. x= 15
C. x= 18
D. x= 9​

Answers

Answer 1

Answer:

X+9=24

Or,x=24-9

:.x=15

Step-by-step explanation:

Answer 2

Answer:

B. x=15

Step-by-step explanation:

To find the solution to the equation, we must get x by itself on one side of the equation.

[tex]x+9=24[/tex]

9 is being added to x. The inverse of addition is subtraction. Subtract 9 from both sides of the equation.

[tex]x+9-9=24-9[/tex]

[tex]x=24-9[/tex]

[tex]x=15[/tex]

Let's check our solution. Plug 15 in for x.

[tex]x+9=24 (x=15)[/tex]

[tex]15+9=24[/tex]

[tex]24=24[/tex]

This checks out, so we know our solution is correct. The answer is B. x=15


Related Questions

Consider the following ordered data. 6 9 9 10 11 11 12 13 14 (a) Find the low, Q1, median, Q3, and high. low Q1 median Q3 high (b) Find the interquartile range.

Answers

Answer:

Low             Q1                Median              Q3                 High

6                  9                     11                      12.5                14

The interquartile range = 3.5

Step-by-step explanation:

Given that:

Consider the following ordered data. 6 9 9 10 11 11 12 13 14

From the above dataset, the highest value = 14  and the lowest value = 6

The median is the middle number = 11

For Q1, i.e the median  of the lower half

we have the ordered data = 6, 9, 9, 10

here , we have to values as the middle number , n order to determine the median, the mean will be the mean average of the two middle numbers.

i.e

median = [tex]\dfrac{9+9}{2}[/tex]

median = [tex]\dfrac{18}{2}[/tex]

median = 9

Q3, i.e median of the upper half

we have the ordered data = 11 12 13 14

The same use case is applicable here.

Median = [tex]\dfrac{12+13}{2}[/tex]

Median = [tex]\dfrac{25}{2}[/tex]

Median = 12.5

Low             Q1                Median              Q3                 High

6                  9                     11                      12.5                14

The interquartile range = Q3 - Q1

The interquartile range =  12.5 - 9

The interquartile range = 3.5

What is 5 feet and 11 inches in inches

Answers

Answer:

60

Step-by-step explanation:

5 is 60 inch

How many times does 1/4 go into 3/8

Answers

Answer:

3/2

Step-by-step explanation:

3/8 ÷ 1/4

Copy dot flip

3/8 * 4/1

12/8

Divide top and bottom by 4

3/2

How do you find x when knowing the probability?

Answers

Answer:

x

Step-by-step explanation:

probability is the branch of mathematics concerning numeral descriptions of how likely an event is to occur or how likely it is that a proposition is true

Which is a perfect square? 6 Superscript 1 6 squared 6 cubed 6 Superscript 5 What is the length of the hypotenuse, x, if (20, 21, x) is a Pythagorean triple

Answers

Answer:

Step-by-step explanation:

Hello, by definition a perfect square can be written as [tex]a^2[/tex] where a in a positive integer.

So, to answer the first question, [tex]6^2[/tex] is a perfect square.

(a,b,c) is a Pythagorean triple means the following

[tex]a^2+b^2=c^2[/tex]

Here, it means that

[tex]x^2=20^2+21^2=841=29^2 \ \ \ so\\\\x=29[/tex]

Thank you.

Answer:

Its B

Step-by-step explanation:

PLEASE HELP!!!
Evaluate the expression when b=4 and y= -3
-b+2y

Answers

Answer: -10

Step-by-step explanation: All you have to do is plug the values into the equation. -4+2(-3). Then you solve the equation using PEDMAS.

1. -4+2(-3)

2. -4+(-6)

3.-4-6

4.-10

Answer:

8

Step-by-step explanation:

-b + 2y

if

b = 4

and

y = 3

then:

-b + 2y = -4 + 2*6 = -4 + 12

= 8

Brian needs to paint a logo using two right triangles. The dimensions of the logo are shown below. What is the difference between the area of the large triangle and the area of the small triangle? ​

Answers

Answer:

7.5 cm²

Step-by-step explanation:

Dimensions of the large ∆:

[tex] base (b) = 3cm, height (h) = 9cm [/tex]

[tex] Area = 0.5*b*h = 0.5*3*9 = 13.5 cm^2 [/tex]

Dimensions of the small ∆:

[tex] base (b) = 2cm, height (h) = 6cm [/tex]

[tex] Area = 0.5*b*h = 0.5*2*6 = 6 cm^2 [/tex]

Difference between the area of the large and the small ∆ = 13.5 - 6 = 7.5 cm²

100 students are interviewed to see which of biology, chemistry or physics they prefer.
59 of the students are girls. 35 of the girls like biology best.
2 of the boys prefer physics.
6 out of the 30 who prefer chemistry are girls.
What percentage of the students prefer biology?

Answers

Answer:

50%

Step-by-step explanation:

Girls                                            Boys

total:                 59                     total:                              41

- Chemistry       35                   - Physics                           2

                       = 24                                    =                     39

                                                 - Chemistry ( 30 - 6 )      24

                                                                   =                     15

Total boys and girls for Biology = 35 + 15 = 50

% = 50/100*100

   = 50%

Hope it helps and also mark it as brainliest!!!!

write a thirdthird-degree polynomial expression that has only two terms with a leading term that has a coefficient of five and a constant of negative two ​

Answers

Answer:

5x^3-2

[tex]ax^{3} +bx^{2} +cx+d\\5x^{3}-given\\ d=-2-given\\5x^{3} -2[/tex]

Answer: [tex]5x^3 - 2[/tex]

Explanation:

The two terms are [tex]5x^3[/tex] and [tex]2[/tex]. Terms are separated by either a plus or minus.

We can write it as [tex]5x^3+(-2)[/tex] which is an equivalent form. Here the two terms are [tex]5x^3[/tex] and [tex]-2[/tex]. This is because adding a negative is the same as subtracting.

The coefficient is the number to the left of the variable.

The degree is the largest exponent, which helps form the leading term.

The third degree polynomial written above is considered a cubic binomial. "Cubic"  refers to the third degree, while "binomial" means there are 2 terms.

We can write something like [tex]5x^3[/tex] as 5x^3 when it comes to computer settings.

Find the Vertical asymptotes of the graph of f
[tex]f(x) = \frac{x + 2}{ {x}^{2} - 4}[/tex]

Answers

Answer:

x = 2 and x = -2

Step-by-step explanation:

To find the vertical asymptotes, set the denominator equal to zero and solve for x:

vertical asymptotes are x = 2 and x = -2

A standardized​ exam's scores are normally distributed. In a recent​ year, the mean test score was and the standard deviation was . The test scores of four students selected at random are ​, ​, ​, and . Find the​ z-scores that correspond to each value and determine whether any of the values are unusual. The​ z-score for is nothing. ​(Round to two decimal places as​ needed.) The​ z-score for is nothing. ​(Round to two decimal places as​ needed.) The​ z-score for is nothing. ​(Round to two decimal places as​ needed.) The​ z-score for is nothing. ​(Round to two decimal places as​ needed.) Which​ values, if​ any, are​ unusual? Select the correct choice below​ and, if​ necessary, fill in the answer box within your choice. A. The unusual​ value(s) is/are nothing. ​(Use a comma to separate answers as​ needed.) B. None of the values are unusual.

Answers

Answer:

The​ z-score for 1880 is 1.34.

The​ z-score for 1190 is -0.88.

The​ z-score for 2130 is 2.15.

The​ z-score for 1350 is -0.37.

And the z-score of 2130 is considered to be unusual.

Step-by-step explanation:

The complete question is: A standardized​ exam's scores are normally distributed. In recent​ years, the mean test score was 1464 and the standard deviation was 310. The test scores of four students selected at random are ​1880, 1190​, 2130​, and 1350. Find the​ z-scores that correspond to each value and determine whether any of the values are unusual. The​ z-score for 1880 is nothing. ​(Round to two decimal places as​ needed.) The​ z-score for 1190 is nothing. ​(Round to two decimal places as​ needed.) The​ z-score for 2130 is nothing. ​(Round to two decimal places as​ needed.) The​ z-score for 1350 is nothing. ​(Round to two decimal places as​ needed.) Which​ values, if​ any, are​ unusual? Select the correct choice below​ and, if​ necessary, fill in the answer box within your choice. A. The unusual​ value(s) is/are nothing. ​(Use a comma to separate answers as​ needed.) B. None of the values are unusual.

We are given that the mean test score was 1464 and the standard deviation was 310.

Let X = standardized​ exam's scores

The z-score probability distribution for the normal distribution is given by;

                          Z  =  [tex]\frac{X-\mu}{\sigma}[/tex]  ~ N(0,1)

where, [tex]\mu[/tex] = mean test score = 1464

           [tex]\sigma[/tex] = standard deviation = 310

S, X ~ Normal([tex]\mu=1464, \sigma^{2} = 310^{2}[/tex])

Now, the test scores of four students selected at random are ​1880, 1190​, 2130​, and 1350.

So, the z-score of 1880 =  [tex]\frac{X-\mu}{\sigma}[/tex]

                                      =  [tex]\frac{1880-1464}{310}[/tex]  = 1.34

The z-score of 1190 =  [tex]\frac{X-\mu}{\sigma}[/tex]

                                =  [tex]\frac{1190-1464}{310}[/tex]  = -0.88

The z-score of 2130 =  [tex]\frac{X-\mu}{\sigma}[/tex]

                                =  [tex]\frac{2130-1464}{310}[/tex]  = 2.15

The z-score of 1350 =  [tex]\frac{X-\mu}{\sigma}[/tex]

                                =  [tex]\frac{1350-1464}{310}[/tex]  = -0.37

Now, the values whose z-score is less than -1.96 or higher than 1.96 are considered to be unusual.

According to our z-scores, only the z-score of 2130 is considered to be unusual as all other z-scores lie within the range of -1.96 and 1.96.

Which of the following is an arithmetic sequence? A.-2, 4, -6, 8, ... B.2, 4, 8, 16, ... C.-8, -6, -4, -2, ...

Answers

Answer:

C. -8, -6, -4, -2, ...

Step-by-step explanation:

An arithmetic sequence increases by the same amount every time through addition or subtraction. There is a common difference.

A: -2, 4, -6, 8, ... If there were a common difference, the numbers would not switch between being positive and back to negative. The numbers would either keep going positive or keep going negative.

B: 2, 4, 8, 16, ... The common difference between 16 and 8 is 16 - 8 = 8. The difference between 8 and 4 is 8 - 4 = 4. Since the difference changes between the numbers, this is not an arithmetic sequence.

C. -8, -6, -4, -2, ... The common difference between -2 and -4 is -2 - (-4) = -2 + 4 = 2. The difference between -4 and -6 is -4 - (-6) = -4 + 6 = 2. The difference between -6 and -8 is -6 - (-8) = -6 + 8 = 2. Since the common difference is always two, this is an arithmetic sequence.

Hope this helps!

What is the difference? Complete the equation. -1 2/5 - (-4/5) = ?

Answers

Answer:

First convert them which will be

-7/5 - (-4/5)

so when you subtract a negative number from negative number they actually subtract ex = -4-(-2) = -2

so its simply 7/5-4/5 then add a negative sign

so

3/5

now add negative sign so

-3/5


What is the volume of a cube with a side length of
of a unit?

Answers

It’s d times it three times length width height

if f(x)=3x-3 and g(x)=-x2+4,then f(2)-g(-2)=

Answers

Answer:

3

Step-by-step explanation:

f(x)=3x-3

g(x)=-x^2+4,

f(2) = 3(2) -3 = 6-3 =3

g(-2) = -(-2)^2+4 = -4+4 = 0

f(2)-g(-2)= = 3-0 = 3

Find the rectangular coordinates of the point with the given polar coordinates.

Answers

Answer:

[tex]( - \sqrt{3} \: an d \: 1)[/tex]

What is the result of question?

Answers

Answer: B. 26x+270 less than or equal to 1,325

Explanation:
26, people attending
x, money spent in lunch for each guest
270, cost of renting the meeting room
Less than or equal to, because 1,325 is the max budget
1,325, is the budget


Hope this helps <3

Answer:

B

Step-by-step explanation:

x can not be greater than (1,325-270)/26 because $270 is fixed for the rental

What is the solution to the following system of equations? 3x-2y=12 6x - 4y = 24

Answers

Answer:

D question,somewhat confusing, itsit's like simultaneous equation,but values are different

Answer:

x = 4 + 2y/3

Step-by-step explanation:

Solve for 2 in the diagram below.
120°
32°
T=

Answers

Step-by-step explanation:

Hello, there!!!

It's so simple here,

Here,

we have is 1 angle is 120°and other is 3x°.

now,

3x°=120° {because when two st.line intersects eachother then the opposite angle formed are equal}

so, 3x°=120

or, x=120°/3

=40°

Therefore, the value of x is 40°.

Hope it helps....

solve for x: 5x+3+8x-4=90

Answers

Answer:

[tex]x = 7[/tex]

Step-by-step explanation:

We can solve the equation [tex]5x+3+8x-4=90[/tex] by isolating the variable x on one side. To do this, we must simplify the equation.

[tex]5x+3+8x-4=90[/tex]

Combine like terms:

[tex]13x - 1 = 90[/tex]

Add 1 to both sides:

[tex]13x = 91[/tex]

Divide both sides by 13:

[tex]x = 7[/tex]

Hope this helped!

Answer:

x = 7

Step-by-step exxplanation:

5x + 3 + 8x - 4 = 90

5x + 8x = 90 - 3 + 4

13x = 91

x = 91/13

x = 7

probe:

5*7 + 3 + 8*7 - 4 = 90

35 + 3 + 56 - 4 = 90

. A discount brokerage selected a random sample of 64 customers and reviewed the value of their accounts. The mean was $32,000 with a population standard deviation of $8,200. What is a 90% confidence interval for the mean account value of the population of customers

Answers

Answer:

The  90% confidence interval is  [tex]\$ \ 30313.9< \mu < \$ \ 33686.13[/tex]

Step-by-step explanation:

From the question we are told that

   The  sample size is  n =  64

     The sample  mean is  [tex]\= x = \$ 32, 000[/tex]

     The  standard deviation is  [tex]\sigma= \$ 8, 200[/tex]

     

Given that the confidence interval is  90% then the level of significance is mathematically evaluated as

             [tex]\alpha = 100 - 90[/tex]

             [tex]\alpha = 10 \%[/tex]

            [tex]\alpha = 0.10[/tex]

Next we obtain the critical value of  [tex]\frac{ \alpha }{2}[/tex] from the normal distribution table , the value is  

       [tex]Z_{\frac{\alpha }{2} } = 1.645[/tex]

  Generally the margin of error is mathematically represented as

        [tex]E = Z_{\frac{\alpha }{2} } * \frac{ \sigma }{ \sqrt{n} }[/tex]

  =>   [tex]E = 1.645 * \frac{ 8200 }{ \sqrt{64} }[/tex]

  =>   [tex]E = 1686.13[/tex]

The 90% confidence interval is mathematically represented as

      [tex]\= x - E < \mu < \= x + E[/tex]

 =>    [tex]32000 - 1689.13 < \mu < 32000 + 1689.13[/tex]

=>    [tex]\$ \ 30313.9< \mu < \$ \ 33686.13[/tex]

Determine which is the appropriate approach for conducting a hypothesis test. ​Claim: The mean RDA of sodium is 2400mg. Sample​ data: n​150, ​3400, s550. The sample data appear to come from a normally distributed population.

Answers

Answer:

Use the student t distribution

Step-by-step explanation:

Here is the formula

t = (x - u) ÷(s/√N)

From the information we have in the question:

n = 150

s = 550

x = 3400

u = mean = 2400

= 3400 - 2400÷ 500/√150

= 1000/44.9

= 22.27

At 0.05 significance level, df = 149 so t tabulated will be 1.65.

We cannot use normal distribution since we do not have population standard deviationWe cannot use normal distribution since we do not have population standard deviationChisquare cannot be used since we are not testing for population varianceWe cannot use normal distribution since we do not have population standard deviationChisquare cannot be used since we are not testing for population varianceThe parametric or bootstrap method cannot be used either.

Gina, Sam, and Robby all rented movies from the same video store. They each rented some dramas, comedies, and documentaries. Gina rented 11 movies total. Sam rented twice as many dramas, three times as many comedies, and twice as many documentaries as Gina. He rented 27 movies total. If Robby rented 19 movies total with the same number of dramas, twice as many comedies, and twice as many documentaries as Gina, how many movies of each type did Gina rent?

Answers

Hi there! :)

Answer:

Gina rented 3 dramas, 5 comedies, and 3 documentaries.

Step-by-step explanation:

To solve, we will need to set up a system of equations:

Let x = # of dramas, y = # of comedies, and z = # of documentaries:

Write equations to represent each person:

Gina:

x + y + z = 11

Sam:

2x + 3y + 2z = 27

Robby:

x + 2y + 2z = 19

Write the system:

x + y + z = 11

2x + 3y + 2z = 27

x + 2y + 2z = 19

Begin by subtracting the third equation from the second:

2x + 3y + 2z = 27

x + 2y + 2z = 19

-----------------------

x + y = 8

If x + y = 8, plug this into the first equation:

(8) + z = 11

z = 11 - 8

z = 3

We found the # of documentaries Gina rented, now we must solve for the other variables:

Subtract the top equation from the third. Substitute in the value of z we solved for:

x + 2y + 2(3) = 19

x + y + (3) = 11

-------------------------

y + 3 = 8

y = 5

Substitute in the values for y and z to solve for x:

x + 5 + 3 = 11

x + 8 = 11

x = 11 - 8

x = 3.

Therefore, Gina rented 3 dramas, 5 comedies, and 3 documentaries.

Answer:

B- x + y + z = 11

2x + 3y + 2z = 27

x + 2y + 2z = 19

Step-by-step explanation:

I took the quiz

A maker of microwave ovens advertises that no more than 10% of its microwaves need repair during the first 5 years of use. In a random sample of 50 microwaves that are 5 years old, 12% needed repairs at a=.04 can you reject the makers claim that no more than 10% of its microwaves need repair during the first five years of use?

Answers

Answer:

We conclude that no more than 10% of its microwaves need repair during the first five years of use.

Step-by-step explanation:

We are given that a maker of microwave ovens advertises that no more than 10% of its microwaves need repair during the first 5 years of use.

In a random sample of 50 microwaves that are 5 years old, 12% needed repairs.

Let p = population proportion of microwaves who need repair during the first five years of use.

So, Null Hypothesis, [tex]H_0[/tex] : p [tex]\leq[/tex] 10%      {means that no more than 10% of its microwaves need repair during the first five years of use}

Alternate Hypothesis, [tex]H_A[/tex] : p > 10%     {means that more than 10% of its microwaves need repair during the first five years of use}

The test statistics that will be used here is One-sample z-test for proportions;

                        T.S.  =  [tex]\frac{\hat p-p}{\sqrt{\frac{p(1-p)}{n} } }[/tex]  ~ N(0,1)

where, [tex]\hat p[/tex] = sample proportion of microwaves who need repair during the first 5 years of use = 12%

           n = sample of microwaves = 50

So, the test statistics =  [tex]\frac{0.12-0.10}{\sqrt{\frac{0.10(1-0.10)}{50} } }[/tex]

                                    =  0.471

The value of z-test statistics is 0.471.

Now, at a 0.04 level of significance, the z table gives a critical value of 1.751 for the right-tailed test.

Since the value of our test statistics is less than the critical value of z as 0.471 < 1.751, so we have insufficient evidence to reject our null hypothesis as the test statistics will not fall in the rejection region.

Therefore, we conclude that no more than 10% of its microwaves need repair during the first five years of use.

Find the area of the shaded regions:

Answers

area of Arc subtending [tex]360^{\circ}[/tex] (i.e. the whole circle) is $\pi r^2$

so area of Arc subtending $\theta^{\circ}$ is, $\frac{ \pi r^2}{360^{\circ}}\times \theta^{\circ}$

$\theta =72^{\circ}$ so the area enclosed by one such arc is $\frac{\pi (10)^272}{360}$

abd there are 2 such arcs, so double the area.

[tex] \LARGE{ \underline{ \boxed{ \rm{ \purple{Solution}}}}}[/tex]

Given:-Radius of the circle = 10 inchesAngle of each sector = 72°Number of sectors = 2

To FinD:-Find the area of the shaded regions....?

How to solve?

For solving this question, Let's know how to find the area of a sector in a circle?

[tex] \large{ \boxed{ \rm{area \: of \: sector = \frac{\theta}{360} \times \pi {r}^{2} }}}[/tex]

Here, Θ is the angle of sector and r is the radius of the circle. So, let's solve this question.

Solution:-

We have,

No. of sectors = 2Angle of sector = 72°

By using formula,

⇛ Area of shaded region = 2 × Area of each sector

⇛ Area of shaded region = 2 × Θ/360° × πr²

⇛ Area of shaded region = 2 × 72°/360° × 22/7 × 10²

⇛ Area of shaded region = 2/5 × 100 × 22/7

⇛ Area of shaded region = 40 × 22/7

⇛ Area of shaded region = 880/7 inch. sq.

⇛ Area of shaded region = 125.71 inch. sq.

☄ Your Required answer is 125.71 inch. sq(approx.)

━━━━━━━━━━━━━━━━━━━━

Reduce the following fraction to lowest terms: 8/14

Answers

Answer:

4/7

Step-by-step explanation:

divide both by two for its simplest form

Answer:4/7

Step-by-step explanation

Divide both the numerator and denominator by 2

The result for the numerator is 8/2=4

that of the denominator is 14/2=7

Therefore the resultant answer is 4/7

Let f(x)=x+8 and g(x)= x2-6x-7 find f(g2)

Answers

Answer:

-7.

Step-by-step explanation:

g(x) = x^2 - 6x - 7

g(2) = 2^2 - 6(2) - 7

= 4 - 12 - 7

= -8 - 7

= -15

f(x) = x + 8

f(-15) = (-15) + 8

= 8 - 15

= -7

Hope this helps!

The probability density function for random variable W is given as follows: Let x be the 100pth percentile of W and y be the 100(1 – p)th percentile of W, where 0

Answers

Answer:

Step-by-step explanation:

A probability density function (pdf) is used for continuous random variables. That is why p is between 0 and 1 (the two extremes - 0 and 1 - exclusive).

X = 100pth percentile of W

Y = 100(1-p)th percentile of W

Expressing Y as a function of X;

Y = 100(1-p)th = 100th - 100pth

Recall that 100pth is same as X, so substitute;

Y = 100th - X

where 100th = hundredth percentile of W and X = 100pth percentile of W  

In the following equation, when x=3, what is the value of y? -4x + 3y = 12 A. 9 B. 3 C. 0 D. 8 PLZ HURRY IM TIMED WILL MARK BRAINLIEST

Answers

Answer:

[tex]\huge\boxed{y = 8}[/tex]

Step-by-step explanation:

-4x + 3y = 12

Given that x = 3

-4 (3) + 3y = 12

-12 + 3y = 12

Adding 12 to both sides

3y = 12+12

3y = 24

Dividing both sides by 3

y = 8

Answer:

y =8

Step-by-step explanation:

-4x + 3y = 12

Let x = 3

-4(3) +3y = 12

-12+3y = 12

Add 12 to each side

-12+12+3y =12+12

3y =24

Divide each side by 3

3y/3 = 24/3

y =8

Give the domain and range of each relation using set notation​

Answers

Answer:

See below.

Step-by-step explanation:

First, recall the meanings of the domain and range.

The domain is the span of x-values covered by the graph.

And the range is the span of y-values covered by the graph.

1)

So, we have here an absolute value function.

As we can see, the domain of the function is all real numbers because the graph stretches left and right infinitely. Therefore, the domain of the function is:

[tex]\{x|x\in\textbb{R}\}[/tex]

(You are correct!)

For the range, notice how the function stops at y=7. The highest point of the function is (-2,7). There graph doesn't and won't ever reach above y=7. Therefore, the range of the graph is all values less than or equal to 7. In set notation, this is:

[tex]\{y|y\leq 7\}[/tex]

2)

We have here an ellipse.

First, for the domain. We can see the the span of x-values covered by the ellipse is from x=-4 to x=6. In other words, the domain is all values in between these two numbers and including them. Therefore, we can write it as such:

[tex]-4\leq x\leq 6[/tex]

So x is all numbers greater than or equal to -4 but less than or equal to 6. This describes the span of x-values. In set notation, this is:

[tex]\{x|-4\leq x\leq 6\}[/tex]

For the range, we can see that the span of x values covered by the ellipse is from y=-5 to y=1. Just like the domain, we can write it like this:

[tex]-5\leq y\leq 1[/tex]

This represents all the y-values between -5 and 1, including -5 and 1.

In set notation, thi is:

[tex]\{y|-5\leq y\leq 1\}[/tex]

Other Questions
Dtermine the answer to (5) + 4 and explain the steps using a number line Factor completely 2c5 + 44c4 + 242c3. 2c3(c + 11)2 2(c + 11)2 2c3(c + 11)(c 11) 2c3(c2 + 22c + 121) Find the domain and the range of the relation.Find the domain of the relation. Select the correct choice below and fill in the answer box tocomplete your choice.O A. The domain is _(Type your answer in interval notation.)B. The domain is {_}(Type an integer or a fraction. Use a comma to separate answers as needed.)Find the range of the relation. Select the correct choice below and fill in the answer box tocomplete your choice.O A. The range is _(Type an integer or a fraction. Use a comma to separate answers as needed.)OB. The range is {_} Solve for x.13(x-3) = 39x=1x=4x=6x= 10 Henry Crouch's law office has traditionally ordered ink refills 50 units at a time. The firm estimates that carrying cost is 35% of the $12 unit cost and that annual demand is about 235 units per year. The assumptions of the basic EOQ model are thought to apply. For what value of ordering cost would its action be optimal? When a substance's particles are packed together tightly and have a definite shape andvolume...it is said to be in what state?solidgasplasmaliquid Covert the verbal expression into an algebraic expression.The product of 23 and a number x Need HelpPlease Show Work a cone with base radius 7 cm has a volume of 308 cm cube find the vertical height of the cone take 22/7pls now For the function f(x) = 3(x 1)2 + 2, identify the vertex, domain, and range. | | ..... ...... ...... ..... .... .... ...... ........... answer to this qns ....................... The writer wants to introduce the topic of electronegativity with a concrete, casually observable example from the natural world. Which choice best accomplishes this goal Solve 2x+2y=6 and 3x-2y=11 what is the value of -19- (-18)? Capitalism gets its name from the fact that capital resources:_________ a. are mostly treated as private property. b. owned by the state or government. c. given the highest priority in the economy's income distribution. d. in the form of money and financial resources. Sunny Day Manufacturing Company is considering investing in a one-year project that requires an initial investment of $450,000. To do so, it will have to issue new common stock and will incur a flotation cost of 2.00%. At the end of the year, the project is expected to produce a cash inflow of $550,000. The rate of return that Sunny Day expects to earn on its project (net of its flotation costs) is:____________White Lion Homebuilders has a current stock price of $22.35 per share, and is expected to pay a per-share dividend of $2.03 at the end of next year. The company's earnings' and dividends' growth rate are expected to grow at the constant rate of 8.70% into the foreseeable future. If White Lion expects to incur flotation costs of 5.00% of the value of its newly-raised equity funds, then the flotation-adjusted (net) cost of its new common stock (rounded to two decimal places) should be:_________Sunny Day Manufacturing Company Co.'s addition to earnings for this year is expected to be $420,000. Its target capital structure consists of 50% debt, 5% preferred, and 45% equity. Determine Sunny Day Manufacturing Company's retained earnings breakpoint: ___________a. $840,000 b. $980,000 c. $933,333 d. $886,666 a pupil performed an experiment in a school lab to show the action of a digestive enzyme on a food substancea) Name and enzyme suitable for such an experiment b) Name a food substance on which the enzyme that you have named will actc) Describe any preparation of the food required before the experiment is performed. If no preparation is required state why? d) give the temperature at which the enzyme-food mix should be maintained for the experiment to work e) how much time is needed for digestion of food in this experiment? f) describe a test to confirm that digestion has occurred Solve for W.W/9 = g Which phrase best describes the relationship indicates by the scatter plotting? Please answer now question